You are on page 1of 12

PC235 Winter 2013

Classical Mechanics

Assignment #4 Solutions
#1 (10 points) JRT Prob. 4.28
Consider a mass m on the end of a spring of force constant k and constrained
to move along the horizontal x axis. If we place the origin at the springs equilibrium
position, the potential energy is 21 kx2 . At time t = 0 the mass is sitting at the
origin and is given a sudden kick to the right so that it moves out to a maximum
displacement xmax = A and then continues to oscillate about the origin.
(a) Write down the equation for conservation of energy and solve it to give the
masss velocity x in terms of the position x and the total energy E.
(b) Show that E = 21 kA2 , and use this to eliminate E from your expression for x.

Find the time for the mass to move from the origin out to a position x.
(c) Solve the result of part (b) to give x as a functionp
of t and show that the mass
executes simple harmonic motion with period 2 m/k.

Solution

(a) Since E = T + U = 12 mx 2 + 21 kx2 , it follows that x(x)

2/m

E 12 kx2 .

(b) At the end point x = A, we know that the kinetic energy T = 0, so


here, E =
1
2
kA
.
Substituting
this
into
the
result
of
part
(a),
we
get
x(x)

A2 x 2 ,
2
p
where = k/m. From the hint given in the problem statement, we find
Z x
Z
1 x
dx
dx

.
(1)
=
t=
)
0
A2 x2
0 x(x
The integral can be evaluated with the substitution x = A sin , and gives
sin1 (x/A). Thus, t = (1/) sin1 (x/A).

(c) Solving for x, we find x(t) = A sin t. x is a sinusoidal function of time,


which is the definition of simple harmonic motion.
For SHM, we know that
p
the period T = 2/, which evaluates to 2 m/k as required.
#2 (10 points) JRT Prob. 4.30
Fig. 4.25 of the text shows a childs toy, which has the shape of a cylinder
mounted on top of a hemisphere. The radius of the hemisphere is R and the CM of
the whole toy is at a height h above the floor.
1

(a) Write down the gravitational potential energy when the toy is tipped to an
angle from the vertical.
(b) For what values of R and h is the equilibrium at = 0 stable?
Solution
(a) As the toy tips, the hemisphere rolls but its center O remains at a fixed height.
However, the height of the CM above O changes from h R to (h R) cos .
Therefore, the PE of the toy is now U () = mg [R + (h R) cos ].
(b) Since dU/d = mg(h R) sin , which vanishes at = 0, we see that the
upright position is an equilibrium, which was probably easy to guess. Next,
d2 U/d2 = mg(h R) cos = mg(R h) at = 0. This tells us that the
equilibrium is only stable if R > h.

#3 (10 points) JRT Prob. 4.36


A metal ball (mass m) with a hole through it is threaded on a frictionless
vertical rod. A massless, non-extensible string of finite length , attached to the ball,
runs over a massless, frictionless pulley and supports a block of mass M , as shown
in the figure below; mass M always hangs vertically downward from the pulley, it
is not free to swing back and forth. For ease of calculations, assume that the pulley
and the ball have a negligible radius. Observe that the positions of the two masses
can be specified by a single coordinate, the angle .
(a) Write down the potential energy U ()
(b) By differentiating U (), find whether the system has an equilibrium position,
and for what values of m and M equilibrium can occur. Discuss the stability
of any equilibrium positions.
Solution
(a) From the figure, we see that h = b/ tan and H = b/ sin . Therefore,


m
gb
M

=
(M m cos ) + const. (2)
U = mgh M gH = gb
sin tan
sin
(b) Equilibrium points exist wherever dU/d = 0. This derivative is (using the
product rule)
dU
gb(m M cos )
m sin2 cos (M m cos )
=
.
= gb
2
d
sin
sin2

(3)

If M < m, dU/d can never equal zero. Therefore, there are no equilibrium
points for M < m.
2

Fig. 1: Configuration for problem #3

If M = m, dU/d = 0 only when = 0. This is an impossible situation;


it would require that the string be infinitely long (which is contradicted in the
problem statement). Therefore, there are no equilibrium points for M = m.
If M > m, dU/d = 0 for 0 = arccos(m/M ). Because of the physical constraint that 0 < 90 , there is only valid equilibrium point. To determine
whether this is a stable or unstable equilibrium, we generally look at the sign
of d2 U/d2 . However, this second derivative is a bit ugly (but certainly not
impossible). Instead, we simply note that, since 0 < 90 , we are always in a regime where cos decreases as increases. Therefore, the factor
(m M cos ) is negative for < 0 and positive for > 0 . Therefore, U ()
has a minimum at 0 , and the equilibrium is stable.

#4 (10 points) JRT Prob. 4.37(a-b)


The figure below shows a massless wheel of radius R, mounted on a frictionless
horizontal axle. A point mass M is glued to the edge of the wheel and a mass m
hangs from a string wrapped around the perimeter of the wheel.
(a) Write down the total PE of the two masses as a function of the angle .
(b) Use this to find the values of m and M for which there are any positions of
equilibrium. Describe the equilibrium positions, discuss their stability, and
explain your answers in terms of torques.
Solution
3

Fig. 2: Configuration for question #4

(a) We will choose the zero-level of PE where = 0. As the wheel turns through
angle , the mass m descends by R (the length of the arc of radius R and
angle .) At the same time, the mass M rises by R(1 cos ); this can be
seen on the figure - draw a horizontal line from the position of M at angle
to intersect the vertical line indicating = 0...the length of the line between
this intersect and the center of the wheel is R cos . The distance between the
bottom of the wheel and the center of the wheel is R. Thus, the height gained
by M while turning through angle is R(1 cos ). Therefore the total PE
is
U () = M gR(1 cos ) mgR.
(4)
(b) The condition for equilibrium is that dU/d = 0. In this case, we require that
M gR sin = mgR, or sin = m/M . Because m and M are real, there are
no positions of equilibrium if m > M . If m < M , there are two points of
equilibrium; the two solutions of = sin1 (m/M ) over a full rotation of 2.
These solutions are symmetrically positioned with respect to = /2 - that
is, symmetric with respect to a horizontal line that passes through the axle.
The equation M gR sin = mgR expresses the fact that the clockwise torque
about the axle due to M is balanced by the counterclockwise torque about the
axle due to m. They only balance at the equilibrium points.
The stability of the equilibrium points depends on the second derivative,
d2 U/d2 = M gR cos . This is positive (stable) for the equilibrium point
with < /2, but negative (unstable) for the equilibrium point with > /2.
This can also be explained in terms of torques. For the equilibrium point with
< /2, an increase in pushes M out farther away (horizontally) from the
axle, thus increasing the torque due to M , which returns M to its equilibrium.
For the equilibrium point with > /2, an increase in brings M closer (horizontally) to the axle, reducing the torque, which causes M to move away from
its equilibrium (from the figure, it is easy to see that the torque due to m does
not depend on ).
4

#5 (5 points) JRT Prob. 4.41


A mass m moves in a circular orbit (centered on the origin) in the field of an
attractive central force with potential energy U = krn . Prove the virial theorem,
that T = nU/2.
Solution
Since the mass moves in a circle, the radial component of its acceleration is just ar =
v 2 /r. By Newtons second law, Fr = mv 2 /r. But Fr = dU/dr = nkrn1 .
Comparing these expressions for Fr , we see that mv 2 = nkrn = nU , and hence
T = 12 mv 2 = nU
.
2

#6 (10 points) JRT Prob. 5.4


An unusual pendulum is made by fixing a string to a horizontal cylinder of
radius R, wrapping the string several times around the cylinder, and then tying a
mass m to the loose end. In equilibrium the mass hangs a distance 0 vertically below the edge of the cylinder. Find the potential energy if the pendulum has swung
to an angle from the vertical. Show that for small angles, U has the Hookes law
form U () = const + 21 k2 , in terms of the coordinate . What is the force constant
k?
Solution
The PE is U = mgh, where h is the height of the mass, which can be arbitrarily
scaled; we will measure it down from the level of the cylinders center. To find h,
note first that as the pendulum swings from equilibrium to angle , a length R of
string unwinds from the cylinder. Thus, the length of string away from the cylinder is
AB = (0 +R), and the height BD is BD = (0 +R) cos . Since the height CD =
R sin , we find from subtraction that h = BD CD = 0 cos + R( cos sin ).
Therefore,
U = mgh = mg [0 cos + R( cos sin )].
(5)
If is small, then we can write cos 1 2 /2 and sin , to give



2
1
1
1 2
= mg0 + mgl0 2 = const+ k2 ,
U mg 0 0 + R (1 )
2
2
2
2
(6)
where we have ignored the very small term in 3 . The constant k = mg0 , which is
actually the same as for a simple pendulum of length 0 .

Fig. 3:

geometry of problem #6

#7 (5 points) JRT Prob. 5.8


(a) If a mass m = 0.2 kg is tied to one end of a spring whose force constant
k = 80 N/m and whose other end is held fixed, what are the angular frequency
0 , the frequency f , and the period of its oscillations? (b) If the initial position
and velocity are x0 = 0 and v0 = 40 m/s, what are the constants A and in the
expression x(t) = A cos(t )?
Solution
(a) =

p
k/m = 20 rad/s, f = /2 = 3.2 Hz, and = 1/f = 0.31 s.

(b) Since x0 = 0, A cos() = 0, so = /2. Since vo = A sin = 40 m/s,


must be positive, = /2, and therefore A = vo / = 2 m.

#8 (10 points)
A tennis ball with a small mass m2 sits on top of a basketball with a large
mass m1 , as in the figure below. The bottom of the basketball is a height h above
the ground, and the bottom of the tennis ball is a height h + d above the ground
(that is, the diameter of the basketball is d. The balls are dropped from this configuration. To what height does the tennis ball bounce?
Hint: Use the results of Problem 4.47 (which you dont need to prove). Work in the
approximation where m1 is much larger than m2 , and assume that the balls bounce
elastically - in this case, the answer doesnt depend on the masses. Also assume, to
aid in visualizing the moment of impact, that the balls are initially separated by a
very small but finite distance and that they bounce instantaneously. If youve never
seen this experiment carried out before, then the answer might surprise you. Try it
at home, just make sure that the two balls are vastly different in mass, that they
both bounce well, and that you dont do it under an expensive chandelier.
Solution
Right before the basketball hits the ground, both balls are moving downward with
6

Fig. 4: Configuration for problem #8

a speed (using 12 mv 2 = mgh by conservation of energy)


v=

2gh.

(7)

Immediately after the basketball hits the ground, it moves upward with speed v
(required since this is an elastic collision), while the tennis ball moves downward
with speed v (remember, they are separated by a very small but finite distance.)
The relative speed (in the frame of the basketball) is therefore 2v (downward.) From
Problem 4.47, this means that after the basketball and tennis ball collide, the tennis
ball will have a velocity of 2v upward relative to the basketball. Since the upward
speed of the basketball is essentially still v (because it is so much more massive
than the tennis ball,) the tennis ball must have an upward speed 3v following the
collision. By conservation of energy, it then rises to a height of H = d + (3v)2 /2g.
But v 2 = 2gh, which means the height is
H = d + 9h.

(8)

#9 (10 points, 2 for each part)


A particle of mass m moves along the positive x-axis in response to a conservative force
4
F (x) = 2 1.
(9)
x
(a) What value of x is a stable equilibrium point for this potential energy function?
Call this point x0 .
(b) If we choose the reference position for potential energy to lie at x = x0 , what
is the potential energy function U (x) corresponding to F (x)?
(c) Express U (x) as a quadratic function of (x x0 ) in the region around x0 .
(d) If the mass oscillates with a small amplitude about x0 , what is the angular
frequency of the oscillations?
7

(e) If the mass is placed at x = 4 and released from rest, what is the turning point
on the other side of x0 ?
Note: The proper units are implied in F (x)...that is, the factor of 4 has units of
Nm2 and the factor of 1 has units of N.
Solution
(a) An equilibrium point is one for which dU/dx = 0, or, more relevantly to us,
where F = 0. Therefore, the point we seek is where x2 = 4, and since the
particle moves only along the positive x-axis, we can say that x0 = 2. We
know that this point is stable, since an increase in x results in a force directed
in the x direction and vice versa.
(b)
U (x) =

F (x)dx = x +

4
+ C,
x

(10)

where C is a constant of integration. Since we require that U (x = 2) = 0, we


have 4 + C = 0, or C = 4. Thus,
U (x) = x +

4
4.
x

(11)

(c) We know that the zeroth- and first-order terms in the Taylor series will both
be zero (since x0 is both an equilibrium point and our reference point for U (x).
Therefore, we only need the second-order term:


1
8
1
1
2
(12)
(x 2)2 = (x 2)2 .
U (x) U (x = 2)(x 2) =
3
2
2 (2)
2
p
(d) From chapter 6, we know that = pk/m. From part (d), we see that k = 1
for this potential, and therefore = 1/m.

(e) We solve this by conservation of mechanical energy (T + U ). At x = 4,


U = 4 + 1 4 = 1 and T = 0 (since the object is at rest). Therefore, the other
turning point will be the other value of x at which U = 1 (since the object is
at rest here as well):
4
4 = 1
x
x2 5x + 4 = 0
x = {1, 4} .
x+

Therefore, the other turning point is x = 1.


8

(13)
(14)
(15)

#10 (10 points) JRT Prob. 5.18


The mass shown in the figure below is resting on a frictionless horizontal table.
Each of the two identical springs has force constant k and unstretched length 0 .
At equilibrium, the mass rests at the origin, and the distances a are not necessarily
equal to 0 (that is, the springs may already be stretched or compressed.) Show
that when the mass moves to a position (x, y), with x and y small, the potential
energy has the form U = 21 (kx x2 + ky y 2 ). Show that if a < 0 , the equilibrium at
the origin is unstable and explain why.

Fig. 5: Configuration for question #9

Solution
When the mass is at position (x, y), the lengths of the two springs are 1 and 2 .
With regards to the figure below, we see that 1 can be approximated as

1/2
p
2x x2 + y 2
2
2
1 =
(a + x) + y = a 1 +
(16)
+
a
a
"


 2 #
1 2x x2 + y 2
1 2x
a 1+

+
2 a
a2
8 a
= a+x+

y2
2a

(a second-degree binomial expansion was used for that second step...we only used
half of the 2nd-order term, since the other half involves fourth-order terms). The
potential energy of spring 1 is therefore
2

1
1
y2
2
U1 =
(17)
k(1 0 ) = k (a 0 ) + x +
2
2
2a

1 
k (a 0 )2 + 2(a 0 )x + x2 + (1 0 /a)y 2

where we have dropped terms of degree higher than 2 in x or y. To find U2 , we only


have to replace x by x, and for the total PE we just add U1 and U2 . When we do
this, the terms that are linear in x cancel, leaving


(18)
U = U1 + U2 = k x2 + (1 0 /a)y 2 + const,
which has, apart from an uninteresting constant, the desired form U = 12 (kx x2 +
ky y 2 ), where kx = 2k and ky = (2 20 /a)k.
9

If a < 0 , ky is negative and the equilibrium at the origin is unstable. This


is because, with a < 0 , the springs are in compression when the mass lies at the
origin. When the mass moves a tiny distance away from the origin along the y axis,
the compression in the springs forces it further away, causing the instability.

Fig. 6: Solution of question #10

#11 (5 BONUS points) JRT Prob. 4.37(c-d)


(c) Plot U () for the cases that m = 0.7M and m = 0.8M , and use your graphs
to describe the behaviour of the system if I release it from rest at = 0.
(d) Find the critical value of m/M on one side of which the system oscillates and
on the other side of which it does not (if released from rest at = 0).
Solution
(c) The required plots are shown below. Consider the behaviour of the wheel
when it is released from = 0. In the case that m = 0.7M , the unstable
hill is at a higher potential energy than that at = 0. Thus, the wheel will
rotate past the stable valley, but will stop before reaching the unstable hill,
and swing back again, oscillating indefinitely between the two turning points.
When m = 0.8M , the unstable hill is at a lower potential energy than that
at = 0. Thus, the wheel will rotate past the stable valley, and then up and
over the hill. will keep increasing indefinitely (or at least until the length of
string holding m runs out.)
(d) As the mass m increases, the tendency for our system to turn in the direction
of increasing increases, and the graph of U () against steadily tilts down
to the right (as is visible in the two figures for part (c)). Somewhere between
these two cases, there must be a critical value mcrit for which the unstable hill
just touches the horizontal axis. To find when this happens, notice that at
this critical value, U and dU/d vanish at the same value of . This gives us
two simultaneous equations:
M (1 cos ) = m
M sin = m
10

[U = 0]
[dU/d = 0] .

(19)
(20)

m=0.7M

m=0.8M

0.5

0.5

0.5

0.5

0.5
[units of ]

0.5
[units of ]

Fig. 7: Plots of U () - question #11

Dividing one of these by the other, we can eliminate both M and m, leaving
an equation for the required angle:
1 cos = sin .

(21)

This equation cant be solved analytically. Numerical root solvers can be used,
or you can simply plot the left- and right-hand sides vs. and see where they
cross. This occurs at 2.33 rad. From the 2nd of the coupled equations,
we know that m = M sin = 0.725M . If you plot U () in this case as in part
(c), you will find that it does indeed represent the critical value that we desire.

#12 (10 BONUS points)


Continuation of #13
Now consider n balls, B1 , B2 , , Bn , having masses m1 , m2 , , mn with m1
m2 mn , standing in a vertical stack (see Fig. 8.) The bottom of the
topmost ball Bn is a height h + above the ground. The balls are dropped. In terms
of n, to what height does the topmost ball bounce? Make the same assumptions as
in question #8. If h = 1 meter, what is the minimum number of balls required so
that the top one bounces to a height of at least 1 kilometer? What is the minimum
number of balls required so that the top one achieves escape velocity ( 11.2 km/s)?
Why is it unreasonable to believe that this would work in practice?
Solution
Right
before B1 hits the ground, all of the balls move downward with speed v =

2gh. We will inductively determine the speed of each ball after it bounces off the
one below it; that is, we wish to ask: if Bi achieves a speed of vi after bouncing off
Bi1 , then what is the speed of Bi+1 after it bounces off B?
The relative speed of Bi+1 and Bi right before they collide is v + vi . This is
11

Fig. 8: Configuration for problem #12

also the relative speed after they bounce. Therefore, since B1 is still moving upward
at essentially speed vi (since it is much more massive than Bi+1 ,) we see that the
final upward speed of Bi+1 equals (v + vi ) + vi = 2vi + v.
Now, regarding the bottom ball B1 , we knows that v1 = v, from which we
can recursively calculate v2 = 3v (which agrees with the results from part (a)),
v3 = 7v, v4 = 15v, etc. In general,
vn = (2n 1)v.

(22)

From conservation of energy, Bn therefore rises to a height of


H =+

[(2n 1) v]2
= + (2n 1)2 h
2g

(where, as in part (a), we used v 2 = 2gh in the last step.)


Rearranging the previous equation, we get
!
r
H
+1 .
n = log2
h

(23)

(24)

Assuming that the stack of balls is dropped from a much lower height than the
desired final height, we can ignore the term . For H=1 km, five balls falls just
short (961 m); six is required, but its a bit of overkill
(3969 m).
Escape velocity is reached when vn vesc = 2gR, where R = 6.4 106 m
is the radius of the earth. Since vn = (2n 1) 2gh, we have
!
r
R
+1 ,
(25)
n log2
h
which means we need n 12. Of course, the forces experienced by the smaller balls
at the top of the pile would be so great in this case that the assumption of elastic
collisions is fairly absurd. Also, its nearly impossible to stack a large number of
balls in a perfectly vertical orientation.
12

You might also like